a我考网

 找回密码
 立即注册

QQ登录

只需一步,快速开始

扫一扫,访问微社区

查看: 95|回复: 0

[考试辅导] GMAT考试写作指导:Argument范文十四

[复制链接]
发表于 2012-8-15 21:50:16 | 显示全部楼层 |阅读模式
74.        In this argument the author predicts a nationwide labor shortage in the near future. ; R2 a; r5 f5 J
The basis for this prediction is an increasing demand for highly skilled workers, ( P/ V1 [% n  F- C, {/ S5 C+ R6 B' S
especially in technical and professional fields, coupled with a slow-growing labor force & A$ Y3 K4 x# J8 x
and a government proposal to cut funds for aid to education. At first glance, the author's
/ M. A/ ]- w) @" \1 L4 w, v2 uargument appears to be somewhat convincing: but further reflection reveals that it is
. I: j+ u* O6 z2 s; \, b$ R5 kbased on some dubious assumptions.
/ d9 p2 ]$ M. Y. B! x      In the first place the author assumes that the present labor force is immobile and
% L9 x. }6 g, ]. j* s3 ythat the demand "for highly skilled workers will have to be met by workers who are * P# o$ \6 h( p% B: B/ X* M
entering the labor market for the first time. Recent American history, however, shows 5 u2 W- `8 s: n
that this assumption is entirely unfounded. At the beginning of the Industrial Revolution 0 R( a; z3 c: K' B2 W
most Americans were farm workers, but by the end of that revolution most had become , b0 J" h4 Q& l0 Q
factory workers. Thus, even though the labor pool remained relatively constant during
3 d4 o0 s! \) w4 K' ]& dthis period, the number of farm workers decreased and the number of factory workers
2 P: j$ U3 S2 U( B6 N  k# D8 K) xincreased. This example clearly demonstrates the mobility of the labor force.. o: v. R+ @1 h+ J9 V" G
      In the second place, the author assumes that the government proposal to cut funds
( O$ s/ {) A9 ~+ a/ Vfor aid to education will have a significant negative impact on the ability to train
3 C4 A6 Y9 R" b, H) p, Uworkers in technical and professional fields. The fact is, however, that the percentage of . v$ u6 V' b( n& o" t
students who rely on government aid for their education is relatively small, so the effect 5 r8 o9 u, |- l1 }( ~4 Q; b  Z
of such cuts would be negligible.
- h) I, i( b" G3 S' w       In conclusion, this argument is unconvincing. To strengthen the argument the ! t2 d" M# D8 E: g
author would have to show that the present work force was relatively static and that the * G5 o4 k3 W2 l- ~$ o' L6 g% [
proposed ours i educational aid would have a deleterious effect on the numbers of high * O  B, S- W, B4 ?) [9 O, |
skilled workers available to enter the work force in the future.
回复

使用道具 举报

您需要登录后才可以回帖 登录 | 立即注册

本版积分规则

Archiver|手机版|小黑屋|Woexam.Com ( 湘ICP备18023104号 )

GMT+8, 2024-5-5 05:45 , Processed in 0.170459 second(s), 21 queries .

Powered by Discuz! X3.4 Licensed

© 2001-2017 Comsenz Inc.

快速回复 返回顶部 返回列表